Last visit was: 24 Apr 2024, 11:57 It is currently 24 Apr 2024, 11:57

Close
GMAT Club Daily Prep
Thank you for using the timer - this advanced tool can estimate your performance and suggest more practice questions. We have subscribed you to Daily Prep Questions via email.

Customized
for You

we will pick new questions that match your level based on your Timer History

Track
Your Progress

every week, we’ll send you an estimated GMAT score based on your performance

Practice
Pays

we will pick new questions that match your level based on your Timer History
Not interested in getting valuable practice questions and articles delivered to your email? No problem, unsubscribe here.
Close
Request Expert Reply
Confirm Cancel
SORT BY:
Date
Tags:
Difficulty: 605-655 Levelx   Weakenx               
Show Tags
Hide Tags
Current Student
Joined: 26 May 2019
Posts: 737
Own Kudos [?]: 263 [1]
Given Kudos: 84
Location: India
GMAT 1: 650 Q46 V34
GMAT 2: 720 Q49 V40
GPA: 2.58
WE:Consulting (Consulting)
Send PM
Intern
Intern
Joined: 12 Nov 2021
Posts: 16
Own Kudos [?]: 1 [0]
Given Kudos: 20
Send PM
GMAT Club Legend
GMAT Club Legend
Joined: 03 Oct 2013
Affiliations: CrackVerbal
Posts: 4946
Own Kudos [?]: 7626 [1]
Given Kudos: 215
Location: India
Send PM
Director
Director
Joined: 20 Apr 2022
Posts: 628
Own Kudos [?]: 254 [0]
Given Kudos: 315
Location: India
GPA: 3.64
Send PM
Re: The proposal to hire ten new police officers in Middletown is quite fo [#permalink]
ThatDudeKnows avigutman why not choice C? if they reduce support for other cities, they will have more money saved as a consequence which they can deploy to use here. So why is this not a weakener? I know the passage and the correct choice but I do not why C is not working out. Assumption also kind of hinges on the fact that funding is not enough for new people. But by reducing support for other services, clearly they can use those resources here .
Tutor
Joined: 11 May 2022
Posts: 1092
Own Kudos [?]: 696 [1]
Given Kudos: 81
Send PM
Re: The proposal to hire ten new police officers in Middletown is quite fo [#permalink]
1
Kudos
Expert Reply
Elite097 wrote:
ThatDudeKnows avigutman why not choice C? if they reduce support for other cities, they will have more money saved as a consequence which they can deploy to use here. So why is this not a weakener? I know the passage and the correct choice but I do not why C is not working out. Assumption also kind of hinges on the fact that funding is not enough for new people. But by reducing support for other services, clearly they can use those resources here .


I don't read C that way. C suggests that the reason you'll need to reduce the other services is because the new police would be eating into the total available funds. You can't allocate those funds to police AND to something else.
Director
Director
Joined: 20 Apr 2022
Posts: 628
Own Kudos [?]: 254 [1]
Given Kudos: 315
Location: India
GPA: 3.64
Send PM
The proposal to hire ten new police officers in Middletown is quite fo [#permalink]
1
Kudos
ThatDudeKnows
No. So the sequence is that they had a certain amount of funds available (say 1000). They say that this amount is sufficient to hire police but not the additional employees for conviction/ prison. Then they say it is foolish to hire more because we only have 1000 for these people and noone else. Fair. Now what we don't know is that they are also reducing funds elsewhere. Choice C tells us that we are reducing services elsewhere. So, once we know they are reducing service elsewhwere worth 500, we also get this additional amount so it does cast doubt on the fact that it is foolish to hire more people. Also there is nothing illogical about this thought process. Pls help clarify. Also this does not mean that they will be reducing other services for the new police because they already had sufficient funds for them to begin with. Where am I wrong and why


ThatDudeKnows wrote:
Elite097 wrote:
ThatDudeKnows avigutman why not choice C? if they reduce support for other cities, they will have more money saved as a consequence which they can deploy to use here. So why is this not a weakener? I know the passage and the correct choice but I do not why C is not working out. Assumption also kind of hinges on the fact that funding is not enough for new people. But by reducing support for other services, clearly they can use those resources here .


I don't read C that way. C suggests that the reason you'll need to reduce the other services is because the new police would be eating into the total available funds. You can't allocate those funds to police AND to something else.
Tutor
Joined: 11 May 2022
Posts: 1092
Own Kudos [?]: 696 [0]
Given Kudos: 81
Send PM
Re: The proposal to hire ten new police officers in Middletown is quite fo [#permalink]
Expert Reply
Elite097 wrote:
ThatDudeKnows
No. So the sequence is that they had a certain amount of funds available (say 1000). They say that this amount is sufficient to hire police but not the additional employees for conviction/ prison. Then they say it is foolish to hire more because we only have 1000 for these people and noone else. Fair. Now what we don't know is that they are also reducing funds elsewhere. Choice C tells us that we are reducing services elsewhere. So, once we know they are reducing service elsewhwere worth 500, we also get this additional amount so it does cast doubt on the fact that it is foolish to hire more people. Also there is nothing illogical about this thought process. Pls help clarify and this does not mean that they will be reducing other services for the new police because they already had sufficient funds for them to begin with.


ThatDudeKnows wrote:
Elite097 wrote:
ThatDudeKnows avigutman why not choice C? if they reduce support for other cities, they will have more money saved as a consequence which they can deploy to use here. So why is this not a weakener? I know the passage and the correct choice but I do not why C is not working out. Assumption also kind of hinges on the fact that funding is not enough for new people. But by reducing support for other services, clearly they can use those resources here .


I don't read C that way. C suggests that the reason you'll need to reduce the other services is because the new police would be eating into the total available funds. You can't allocate those funds to police AND to something else.


Where did you get 500 from. Using your numbers, it seems far more likely (and certainly possible) for that figure to just be the 100 that goes to the additional police. How did you decide that it should be a number greater than the amount that goes to the police.

At the end of the day, I think there’s only so much benefit that can be attained from picking apart an argument, question, or answer choice. You want to make sure that you learn whatever you can in order to learn the type of thinking, but arguing a specific question once you’ve already gotten what you can out of it doesn’t help further improve your score!

Posted from my mobile device
Tutor
Joined: 17 Jul 2019
Posts: 1304
Own Kudos [?]: 2285 [1]
Given Kudos: 66
Location: Canada
GMAT 1: 780 Q51 V45
GMAT 2: 780 Q50 V47
GMAT 3: 770 Q50 V45
Send PM
The proposal to hire ten new police officers in Middletown is quite fo [#permalink]
1
Kudos
Expert Reply
Elite097 wrote:
Why not choice C? if they reduce support for other cities, they will have more money saved as a consequence which they can deploy to use here. So why is this not a weakener? I know the passage and the correct choice but I do not why C is not working out. Assumption also kind of hinges on the fact that funding is not enough for new people. But by reducing support for other services, clearly they can use those resources here .


(C) claims that approving the proposal would mean that support for other city services will have to be reduced during the next fiscal year. To me that seems to strengthen the claim that the proposal is foolish, not weaken it. I'm assuming, of course, that the other city services aren't currently overfunded. If they are, then I suppose (C) neither strengthens nor weakens the conclusion.

I think the error in your thinking, Elite097, is that you're evaluating whether the consequence of approving the proposal (support for other city services will have to be reduced during the next fiscal year) could cause the proposal to be not foolish, but that's circular logic. Let's abstract this reasoning:
X is a foolish proposal because it would lead to problem Y.
Answer choice (C) says: If X is approved, negative impact Z will occur.
If Z occurs, we can avoid problem Y (are you sure?)... Therefore, X isn't foolish after all.
Director
Director
Joined: 20 Apr 2022
Posts: 628
Own Kudos [?]: 254 [1]
Given Kudos: 315
Location: India
GPA: 3.64
Send PM
The proposal to hire ten new police officers in Middletown is quite fo [#permalink]
1
Kudos
I think you did not follow my train of thought so let me clarify. It could be 50 or 100 to. That does not matter. Point is they already had 1000 and that was sufficient to accomodte police but not other employees. We do not know now whether they have another source of revenue too (in this case- reducing costs from other services). Now, in , we are told exactly this that getting funds for police would lead to reducing other services (which means we would get this additional amount of saving too whether it is 50 or 100 or 500 doesnt matter; all we care about is that we have this additional amount from savings too). So altogether we now have 1000 (Originally)+ 100 (from savings) and we already know 1000 is sufficient for the police so we can use this additional 100 for additional hires and hence it is not foolish after all to hire more. ThatDudeKnows
Director
Director
Joined: 20 Apr 2022
Posts: 628
Own Kudos [?]: 254 [1]
Given Kudos: 315
Location: India
GPA: 3.64
Send PM
The proposal to hire ten new police officers in Middletown is quite fo [#permalink]
1
Kudos
Yes i think we can be sure to the extent of cast doubt on the conclusion. Conclusion says they are foolish cuz they have only sufficient funds but if we show them that we have even slightly more, it is enough to CAST doubt which is all we have to do in weaken and not break it completely right? avigutman. I am not sure what you used as a reason to think of it as a strengthener. Probably I am missing some details in what you wrote so if it is something with the verb or sequencing it might help if yu state it explicitly.

avigutman wrote:
Elite097 wrote:
Why not choice C? if they reduce support for other cities, they will have more money saved as a consequence which they can deploy to use here. So why is this not a weakener? I know the passage and the correct choice but I do not why C is not working out. Assumption also kind of hinges on the fact that funding is not enough for new people. But by reducing support for other services, clearly they can use those resources here .


(C) claims that approving the proposal would mean that support for other city services will have to be reduced during the next fiscal year. To me that seems to strengthen the claim that the proposal is foolish, not weaken it. I'm assuming, of course, that the other city services aren't currently overfunded. If they are, then I suppose (C) neither strengthens nor weakens the conclusion.

I think the error in your thinking, Elite097, is that you're evaluating whether the consequence of approving the proposal (support for other city services will have to be reduced during the next fiscal year) could cause the proposal to be not foolish, but that's circular logic. Let's abstract this reasoning:
X is a foolish proposal because it would lead to problem Y.
Answer choice (C) says: If X is approved, negative impact Z will occur.
If Z occurs, we can avoid problem Y (are you sure?)... Therefore, X isn't foolish after all.
Tutor
Joined: 11 May 2022
Posts: 1092
Own Kudos [?]: 696 [0]
Given Kudos: 81
Send PM
Re: The proposal to hire ten new police officers in Middletown is quite fo [#permalink]
Expert Reply
Elite097 wrote:
I think you did not follow my train of thought so let me clarify. It could be 50 or 100 to. That does not matter. Point is they already had 1000 and that was sufficient to accomodte police but not other employees. We do not know now whether they have another source of revenue too (in this case- reducing costs from other services). Now, in , we are told exactly this that getting funds for police would lead to reducing other services (which means we would get this additional amount of saving too whether it is 50 or 100 or 500 doesnt matter; all we care about is that we have this additional amount from savings too). So altogether we now have 1000 (Originally)+ 100 (from savings) and we already know 1000 is sufficient for the police so we can use this additional 100 for additional hires and hence it is not foolish after all to hire more. ThatDudeKnows


Answer choice C: Hiring more police is going to reduce other services.
Argument: Hiring more police is foolish.

Given those two sentences, the road to making C weaken requires at least one additional "if" assumption. You and I disagree, but let's say we don't. Let's say C weakens. The question asks us which answer choice will most seriously weaken.

Per E, we have the money to hire the police, and hiring the police will be a deterrent on crime such that we may have no (or at least less than usual) additional court or prison expense. Wouldn't that make hiring the police a good thing, not "foolish?" Tell me why you eliminated E. If you do a good job with that, we can come back to C. ;)

The biggest take-away here is probably that if you find yourself needing to tell a long story to get to your answer choice, especially if you have an "if" in your story and ESPECIALLY if you have more than one "if," odds are that you've got the wrong answer choice.
Tutor
Joined: 17 Jul 2019
Posts: 1304
Own Kudos [?]: 2285 [2]
Given Kudos: 66
Location: Canada
GMAT 1: 780 Q51 V45
GMAT 2: 780 Q50 V47
GMAT 3: 770 Q50 V45
Send PM
Re: The proposal to hire ten new police officers in Middletown is quite fo [#permalink]
2
Kudos
Expert Reply
Elite097 wrote:
I am not sure what you used as a reason to think of it as a strengthener. Probably I am missing some details in what you wrote so if it is something with the verb or sequencing it might help if yu state it explicitly.

No problem, Elite097, I will try again.
Quote:
If funding for the new police officers’ salaries is approved, support for other city services will have to be reduced during the next fiscal year

This is an 'if' statement. What it tells us is that the proposal is even more foolish than we originally thought: it's not just that we won't be able to afford the salaries of additional court and prison employees... On top of that, we will have to reduce support for other city services.
I believe your confusion comes from a misinterpretation of what answer choice (C) is saying.
I think you're interpreting it in the following way:
(C) if funding for the new police officers' salaries is approved, the city will be able to reduce its spending on other city services during the next fiscal year.
Do you see the subtle difference? If you indeed had this interpretation in mind, you probably figured that thanks to the extra cops, we can save money in other areas, and use the savings as we see fit.
Director
Director
Joined: 20 Apr 2022
Posts: 628
Own Kudos [?]: 254 [1]
Given Kudos: 315
Location: India
GPA: 3.64
Send PM
Re: The proposal to hire ten new police officers in Middletown is quite fo [#permalink]
1
Kudos
ThatDudeKnows since I understand E well and see some merit in it bein correct after getting it wrong, I will take the challenge of telling you why it is wrong so we can go back to C.
Pasaage's conclusion is that it is foolish to hire more police and the premise is that they if they hire more police, they will have to take the burden of paying for additional costs that will come with catching and putting people in to the prison because more cops would mean more catching and hence the assumption is that more cops lead to more catching. E directly attacks that and says that more cops will not lead to more catching and prisons. and hence it is not foolish to hire more plice. I hope this aligns with you so can we please go back to C with the understanding I had given with numerical example ThatDudeKnows

PS I have used some short forms while explaining the scenario so please focus on my broader understanding.


ThatDudeKnows wrote:
Elite097 wrote:
I think you did not follow my train of thought so let me clarify. It could be 50 or 100 to. That does not matter. Point is they already had 1000 and that was sufficient to accomodte police but not other employees. We do not know now whether they have another source of revenue too (in this case- reducing costs from other services). Now, in , we are told exactly this that getting funds for police would lead to reducing other services (which means we would get this additional amount of saving too whether it is 50 or 100 or 500 doesnt matter; all we care about is that we have this additional amount from savings too). So altogether we now have 1000 (Originally)+ 100 (from savings) and we already know 1000 is sufficient for the police so we can use this additional 100 for additional hires and hence it is not foolish after all to hire more. ThatDudeKnows


Answer choice C: Hiring more police is going to reduce other services.
Argument: Hiring more police is foolish.

Given those two sentences, the road to making C weaken requires at least one additional "if" assumption. You and I disagree, but let's say we don't. Let's say C weakens. The question asks us which answer choice will most seriously weaken.

Per E, we have the money to hire the police, and hiring the police will be a deterrent on crime such that we may have no (or at least less than usual) additional court or prison expense. Wouldn't that make hiring the police a good thing, not "foolish?" Tell me why you eliminated E. If you do a good job with that, we can come back to C. ;)

The biggest take-away here is probably that if you find yourself needing to tell a long story to get to your answer choice, especially if you have an "if" in your story and ESPECIALLY if you have more than one "if," odds are that you've got the wrong answer choice.
Director
Director
Joined: 20 Apr 2022
Posts: 628
Own Kudos [?]: 254 [0]
Given Kudos: 315
Location: India
GPA: 3.64
Send PM
The proposal to hire ten new police officers in Middletown is quite fo [#permalink]
avigutman
Thanks. Are you saying that C is saying that IN ORDER TO pay for the additional police officers, the other services will have to be reduced to be able to pay for these police staff and hence reducing other services will not lead to saving money. So given my previous example of 1000 and 500, are we saying that money for these police officers (1000) was not already sufficient from beforehand but IN ORDER TO pay them, we would have to reduce the other services so we are foolish after all? Earlier I had interpreted like we already have 1000 to pay these officers and that the saving will lead to additional 500, let's say, and that won't make our plan foolish. So is my understanding wrong because "will have to reduce" means that they will need to reduce funding of other services to pay for the officers and I interpreted it wrongly as they can reduce other services which will lead to additional savings? So is it something to do with the meaning of tense 'will have to'? Will have to means they will have to do this before using the budget for police officers and not that once budget is approved, this will purposely reduce savings for other services?

avigutman wrote:
Elite097 wrote:
I am not sure what you used as a reason to think of it as a strengthener. Probably I am missing some details in what you wrote so if it is something with the verb or sequencing it might help if yu state it explicitly.

No problem, Elite097, I will try again.
Quote:
If funding for the new police officers’ salaries is approved, support for other city services will have to be reduced during the next fiscal year

This is an 'if' statement. What it tells us is that the proposal is even more foolish than we originally thought: it's not just that we won't be able to afford the salaries of additional court and prison employees... On top of that, we will have to reduce support for other city services.
I believe your confusion comes from a misinterpretation of what answer choice (C) is saying.
I think you're interpreting it in the following way:
(C) if funding for the new police officers' salaries is approved, the city will be able to reduce its spending on other city services during the next fiscal year.
Do you see the subtle difference? If you indeed had this interpretation in mind, you probably figured that thanks to the extra cops, we can save money in other areas, and use the savings as we see fit.

Originally posted by Elite097 on 24 Jun 2022, 22:45.
Last edited by Elite097 on 24 Jun 2022, 23:11, edited 1 time in total.
Tutor
Joined: 11 May 2022
Posts: 1092
Own Kudos [?]: 696 [1]
Given Kudos: 81
Send PM
Re: The proposal to hire ten new police officers in Middletown is quite fo [#permalink]
1
Kudos
Expert Reply
Elite097 wrote:
avigutman
Thanks. Are you saying that C is saying that IN ORDER TO pay for the additional police officers, the other services will have to be reduced to be able to pay for these police staff and hence reducing other services will not lead to saving money. So given my previous example of 1000 and 500, are we saying that money for these police officers (1000) was not already sufficient from beforehand but IN ORDER TO pay them, we would have to reduce the other services so we are foolish after all? Earlier I had interpreted like we already have 1000 to pay these officers and that the saving will lead to additional 500, let's say, and that won't make our plan foolish. So is my understanding wrong because "will have to reduce" means that they will need to reduce funding of other services to pay for the officers and I interpreted it wrongly as they can reduce other services which will lead to additional savings?

avigutman wrote:
Elite097 wrote:
I am not sure what you used as a reason to think of it as a strengthener. Probably I am missing some details in what you wrote so if it is something with the verb or sequencing it might help if yu state it explicitly.

No problem, Elite097, I will try again.
Quote:
If funding for the new police officers’ salaries is approved, support for other city services will have to be reduced during the next fiscal year

This is an 'if' statement. What it tells us is that the proposal is even more foolish than we originally thought: it's not just that we won't be able to afford the salaries of additional court and prison employees... On top of that, we will have to reduce support for other city services.
I believe your confusion comes from a misinterpretation of what answer choice (C) is saying.
I think you're interpreting it in the following way:
(C) if funding for the new police officers' salaries is approved, the city will be able to reduce its spending on other city services during the next fiscal year.
Do you see the subtle difference? If you indeed had this interpretation in mind, you probably figured that thanks to the extra cops, we can save money in other areas, and use the savings as we see fit.



There you go! Avi phrased it well in his last post.
Tutor
Joined: 17 Jul 2019
Posts: 1304
Own Kudos [?]: 2285 [0]
Given Kudos: 66
Location: Canada
GMAT 1: 780 Q51 V45
GMAT 2: 780 Q50 V47
GMAT 3: 770 Q50 V45
Send PM
Re: The proposal to hire ten new police officers in Middletown is quite fo [#permalink]
Expert Reply
Elite097 wrote:
Are you saying that C is saying that IN ORDER TO pay for the additional police officers, the other services will have to be reduced to be able to pay for these police staff and hence reducing other services will not lead to saving money. So given my previous example of 1000 and 500, are we saying that money for these police officers (1000) was not already sufficient from beforehand but IN ORDER TO pay them, we would have to reduce the other services so we are foolish after all? Earlier I had interpreted like we already have 1000 to pay these officers and that the saving will lead to additional 500, let's say, and that won't make our plan foolish. So is my understanding wrong because "will have to reduce" means that they will need to reduce funding of other services to pay for the officers and I interpreted it wrongly as they can reduce other services which will lead to additional savings? So is it something to do with the meaning of tense 'will have to'? Will have to means they will have to do this before using the budget for police officers and not that once budget is approved, this will purposely reduce savings for other services?

Yes, Elite097. I want to make sure that you can apply this lesson to other problems in the future, so let's focus on the general structure of answer choice (C):
Quote:
If funding for the new police officers’ salaries is approved, support for other city services will have to be reduced during the next fiscal year
If X happens, Y will have to happen

See if you can pick out which of the following sentences match this structure, and what makes them different from the ones that don't:
(A) If I win the lottery, I will buy a private jet.
(B) If I lose my voice, I will have to stop singing.
(C) If I crash my car, I will have to walk to work.
(D) If I crash my car, the insurance company will buy me a new one.
Director
Director
Joined: 20 Apr 2022
Posts: 628
Own Kudos [?]: 254 [0]
Given Kudos: 315
Location: India
GPA: 3.64
Send PM
The proposal to hire ten new police officers in Middletown is quite fo [#permalink]
avigutman wrote:
Yes, Elite097. I want to make sure that you can apply this lesson to other problems in the future, so let's focus on the general structure of answer choice (C):
Quote:
If funding for the new police officers’ salaries is approved, support for other city services will have to be reduced during the next fiscal year
If X happens, Y will have to happen

See if you can pick out which of the following sentences match this structure, and what makes them different from the ones that don't:
(A) If I win the lottery, I will buy a private jet-----If X happens, Y will be able to happen ( not similar)
(B) If I lose my voice, I will have to stop singing.--- If X happens, I will be forced to do Y (similar)
(C) If I crash my car, I will have to walk to work. ------If X happens, I will be forced to do Y (similar)
(D) If I crash my car, the insurance company will buy me a new one.-----If X happens, Y will be able to happen ( not similar)


The difference is that in 2 of those cases, the outcome is necessary but in 2 of them, the outcome is possible.
Hence, if the budget is approved, I will be forced to reduce spending on other services to allocate it to police officers so BASICALLY THE FUNDING WILL ACTUALLY GO FROM THE OTHER REDUCED SERVICES' SAVINGS. Without cutting the other services, I will not be able to pay for the police officers. And it does not mean that if the budget is approved, I will be able to reduce other services and increase saving. Pls correct me avigutman
Tutor
Joined: 17 Jul 2019
Posts: 1304
Own Kudos [?]: 2285 [1]
Given Kudos: 66
Location: Canada
GMAT 1: 780 Q51 V45
GMAT 2: 780 Q50 V47
GMAT 3: 770 Q50 V45
Send PM
Re: The proposal to hire ten new police officers in Middletown is quite fo [#permalink]
1
Kudos
Expert Reply
Elite097 wrote:
avigutman wrote:
Yes, Elite097. I want to make sure that you can apply this lesson to other problems in the future, so let's focus on the general structure of answer choice (C):
Quote:
If funding for the new police officers’ salaries is approved, support for other city services will have to be reduced during the next fiscal year
If X happens, Y will have to happen

See if you can pick out which of the following sentences match this structure, and what makes them different from the ones that don't:
(A) If I win the lottery, I will buy a private jet-----If X happens, Y will be able to happen ( not similar)
(B) If I lose my voice, I will have to stop singing.--- If X happens, I will be forced to do Y (similar)
(C) If I crash my car, I will have to walk to work. ------If X happens, I will be forced to do Y (similar)
(D) If I crash my car, the insurance company will buy me a new one.-----If X happens, Y will be able to happen ( not similar)


The difference is that in 2 of those cases, the outcome is necessary but in 2 of them, the outcome is possible.
Hence, if the budget is approved, I will be forced to reduce spending on other services to allocate it to police officers so BASICALLY THE FUNDING WILL ACTUALLY GO FROM THE OTHER REDUCED SERVICES' SAVINGS. Without cutting the other services, I will not be able to pay for the police officers. And it does not mean that if the budget is approved, I will be able to reduce other services and increase saving. Pls correct me avigutman


You got it now, Elite097.
User avatar
Non-Human User
Joined: 01 Oct 2013
Posts: 17213
Own Kudos [?]: 848 [0]
Given Kudos: 0
Send PM
Re: The proposal to hire ten new police officers in Middletown is quite fo [#permalink]
Hello from the GMAT Club VerbalBot!

Thanks to another GMAT Club member, I have just discovered this valuable topic, yet it had no discussion for over a year. I am now bumping it up - doing my job. I think you may find it valuable (esp those replies with Kudos).

Want to see all other topics I dig out? Follow me (click follow button on profile). You will receive a summary of all topics I bump in your profile area as well as via email.
GMAT Club Bot
Re: The proposal to hire ten new police officers in Middletown is quite fo [#permalink]
   1   2 
Moderators:
GMAT Club Verbal Expert
6919 posts
GMAT Club Verbal Expert
238 posts
CR Forum Moderator
832 posts

Powered by phpBB © phpBB Group | Emoji artwork provided by EmojiOne